• Shuffle
    Toggle On
    Toggle Off
  • Alphabetize
    Toggle On
    Toggle Off
  • Front First
    Toggle On
    Toggle Off
  • Both Sides
    Toggle On
    Toggle Off
  • Read
    Toggle On
    Toggle Off
Reading...
Front

Card Range To Study

through

image

Play button

image

Play button

image

Progress

1/75

Click to flip

Use LEFT and RIGHT arrow keys to navigate between flashcards;

Use UP and DOWN arrow keys to flip the card;

H to show hint;

A reads text to speech;

75 Cards in this Set

  • Front
  • Back
  • 3rd side (hint)
The conclusion of the argument follows logically if which one of the following is assumed
Assumption
Which one of the following is an assumption on which Fulan's argument depends
Assumption
Fulana's response depends on the presupposition that
Assumption
The argument above is based on which one of the following assumptions
Assumption
The argument in Fulan's defense depends on the assumption that.
Assumption
Each of the following is an assumption on which the argument depends EXCEPT;
Assumption
The explanation offered by the author of the passage makes the assumption that.
Assumption
Which of the following is assumed by Fulana's argument?
Assumption
The authors conclusion follows logically if which of the following is assumed?
Assumption
The conclusion of the argument is properly drawn if which of the following is assumed.
Assumption
Which one of the following is an assumption on which the argument relies?
Assumption
Which of the following, if assumed, enables the economist's conclusion to be properly drawn?
Assumption
Which of the following assumptions is required by Fulan's argument.?
Assumption
An assumption made in the explanation offered by the author of the passage is that.
Assumption
Which is if true is the logically strongest counter to...?
Weaken
Each of the following, if true support the claim above EXCEPT...?
Weaken
Which of the following if true helps justify the....?
Strengthen
The reasoning in the stimulus would be most strengthened if which of the following were true...?
Strengthen
Which of the following, if true, most undermines the conclusion?
Weaken
Which of the following if true, most seriously weakens the argument?
Weaken
Which one of the following, if true most calls into question the nutritionists argument?
Weaken
Which one of the following, if true most seriously weakens the argument?
Weaken
Which of the following, if true, supports Fulan's hypothesis?
Strengthen
Which of the following, if added to the argument, most helps to justify the authors conclusion?
Strengthen
Which of the following if true, most strongly indicates... What was stated in the (Conclusion)
Strengthen
Which of the following, if true, provides the strongest support for the Fulana's claim...?
Strengthen
Which of the following, if true, casts the most doubt on ...?
Weaken
Which of the following, if true, provides the most support for the reasoning above...?
Strengthen
Each of the following, if true support the claim above EXCEPT...?
Weaken
Which of the following if true helps justify the....?
Strengthen
Which one of the following, if true most calls into question the nutritionists argument?
Weaken
Which one of the following, if true most seriously weakens the argument?
Weaken
Which of the following, if true, supports Fulan's hypothesis?
Strengthen
Which of the following, if added to the argument, most helps to justify the authors conclusion?
Strengthen
Which of the following if true, most strongly indicates... What was stated in the (Conclusion)
Strengthen
Which of the following, if true, provides the strongest support for the Fulana's claim...?
Strengthen
Which of the following, if true, casts the most doubt on ...?
Weaken
Which of the following, if true, provides the most support for the reasoning above...?
Strengthen
The reasoning in the argument is most vulnerable to criticism on the grounds that the argument
Flaw
Fulan's reasoning is most vulnerable to criticism on the grounds that
Flaw
The questionable aspect of the reasoning above is that it...
Flaw
The argument's reasoning is questionable because the argument
Flaw
The reasoning above is questionable because it.
Flaw
Which one of the following could the opponent properly cite as indicating a flaw in the proponents reasoning concerning vitamin losses?
Flaw
Fulan's response most strongly supports the contention that Fulan misunderstood Fulana to be asserting that.
Flaw
Which of the following is the strongest criticism of Fulan's reasoning.
Flaw
The reasoning in the argument is questionable because the argument
Flaw
The reasoning in the argument is flawed because the argument
Flaw
Which one of the following logically completes the argument?
Inference
The Statements above, if true support which one of the following
Inference
If Fulan's statements are true then which one of the following must be true?
Inference
If all of the statements above are true, which one of the following CANNOT be true?
Inference
Fulana's statements, if true, must strongly support which one of the following?
Inference
The above statements, if true, most strongly support which one of the following?
Inference
Which one of the following inferences is most strongly supported by the rules described above?
Inference
Which of the following can be properly inferred from the passage.
Inference
Which of the following most logically completes the passage?
Inference
Which of the following is most supported by the information above?
Inference
Which one of the following can be properly concluded from the passage?
Inference
Each of the following is supported by the information above EXCEPT?
Inference
If the statements above are true, which of the following must be true on the basis of them?
Inference
If all of the statements in the passage are true, each of the following must also be true EXCEPT:
Inference
Which one of the following conclusions is best supported by the passage?
Inference
Which of the following principles is violated by Fulana's reasoning?
Principle
Which one of the principles, if valid, most helps to justify the reasoning above?
Principle
Which of the following conforms most closely to the principle illustrated above?
Principle
The situation described above most closely illustrates which one of the following propositions?
Principle
Which one of the following judgments most closely conforms to the principles cited above?
Principle
Which of the following judgments most closely conforms principle cited above?
Principle
Which of the following ashya most clearly violates ashya cited above?
Principle
Explain what the basics of a Principle question and the different types
On a principle question the stem should tell you one of the following before moving on to the passage: (1)Is the principle in the passage or the stem (2)are you identifying the principle or applying the principle or both.
How are principle questions similar to other questions?
Whereas other questions may deal with specifics, principle questions deal with broad law like rule that can be applied to a variety of specific situations.
"Identifying the Principle" means?
Identify a broad principle in the correct answer choice from a specific scenario in the stimulus.
Which one of the following principles, if valid, most helps to justify the reasoning in the argument.
"Applying the Principle" means?
Apply the broad principle in the stimulus to a specific scenario in the answer choices.
Which one of the following summer weather reports most closely conforms to the principles stated above.
Identify and Apply the principle means?
Identify a principle from as specific scenario in the stimulus and then apply the principle to another specific scenario in the choices.
Which one of the following arguments illustrates a principle most similar to the principle underling the argument above.